Free NCLEX practice review exam questions and answers with reasoning/explanation. Actual NCLEX questions and problems. Free NCLEX tutorial. Free NCLEX practice review exam with video. Different NCLEX practice questions. Automatic. Easy to Use.

Answer questions: START HERE | Psychotic Disorders | Substance Abuse, Eating Disorders, Impulse Control Disorders | OTHERS

Review for NCLEX-RN Examination 140-300

--> QUESTION NUMBER _ 140 _ about (MC)


QUESTION: "The mother of an 8-year-old child with a fluid restriction of 1000 mL/day is staying with the child in the room. Which of the following would be most appropriate for the nurse to include in the child's plan of care?"

CHOICES

( O ) a.) Discussing the fluid restriction with the mother and child, allowing them to decide how to allocate the fluids over the 24 hours.

( X ) b.) Explaining to the mother that hospital personnel will assume the responsibility for providing fluids to the child.

( X ) c.) Letting the child drink fluid until the limit is reached and then allowing the child to drink no more fluids.

( X ) d.) Telling the mother exactly how much fluid the child can have each hour, showing her examples of the amount.


RATIONALE: Planning the child's fluid restriction with the mother and child is most appropriate because the mother and child would best know the child's usual pattern of fluid intake. Doing so also provides the mother with a feeling of some control over her child's situation and helps to promote compliance. Anyone, not just hospital personnel, can provide the child with fluids. However, a strict record of the child's intake must be kept to ensure adherence to the restriction. It is not advisable to allow a client on fluid restriction to drink all the allotted fluid at once. This may result in many thirsty hours for the client. The nurse also should remind the mother to count fluids used when the child takes any medications. Telling the mother exactly how much fluid the child can have each hour restricts the extent of the mother's and child's participation in care. Additionally, doing so ignores the child's usual needs, such as the usual pattern of fluid intake, possibly interfering with adherence to the fluid restriction. NURSING PROCESS STEP: Planning CLIENT NEEDS CATEGORY: Physiological integrity CLIENT NEEDS SUBCATEGORY: Physiological adaptation
******************************

--> QUESTION NUMBER _ 145 _ about (MC)


QUESTION: "When determining the effectiveness of teaching a child's mother about sickle cell disease, which of the following statements by the mother indicates the need for additional teaching? "

CHOICES

( X ) a.) "I've started to give him some extra fluids with and between meals."

( X ) b.) "I'm concerned about how the hospital staff will manage his pain."

( O ) c.) "He's going to be playing on a soccer team when he's feeling better."

( X ) d.) "I've told the child's father that both he and I are carriers of the disease."


RATIONALE: Physical and emotional stress can precipitate a sickle cell crisis. Physical exercise such as running involved in soccer would increase the child's risk for a crisis. Thus, the mother needs additional instructions about this area. Providing extra fluids with and in between meals is appropriate because it is important for the child with sickle cell disease to keep well-hydrated. In addition, these children often have nephrosis related to sickle cell disease and have difficulty conserving fluids. Therefore, they need up to 150% of normal fluid intake. Pain control is an issue in sickle cell crisis. The mother is showing concern for her child by asking how pain will be managed. Sickle cell disease is an autosomal recessive disease. For the child to have the disease, both parents must carry the recessive gene. NURSING PROCESS STEP: Evaluation CLIENT NEEDS CATEGORY: Physiological integrity CLIENT NEEDS SUBCATEGORY: Physiological adaptation
******************************

--> QUESTION NUMBER _ 148 _ about (MC)


QUESTION: "Which of the following would be the priority nursing diagnosis for a 4-week-old infant with a diagnosis of pyloric stenosis?"

CHOICES

( X ) a.) Constipation.

( O ) b.) Deficient Fluid Volume.

( X ) c.) Imbalanced Nutrition, less than body requirements.

( X ) d.) Impaired Swallowing.


RATIONALE: Infants with pyloric stenosis generally have a history of spitting up, which progresses to projectile vomiting, weight loss, decrease in number of stools, and some degree of dehydration. Infants with dehydration need fluid and electrolyte replacement before surgery. A decrease in the number of stools, not constipation, is associated with pyloric stenosis. Although the infant's nutrition may be affected, a fluid volume deficit is the priority. Infants have a greater percentage of water per body weight and are at high risk for fluid imbalances. Pyloric stenosis is not associated with difficulty in swallowing. NURSING PROCESS STEP: Analysis, nursing diagnosis CLIENT NEEDS CATEGORY: Physiological integrity CLIENT NEEDS SUBCATEGORY: Physiological adaptation
******************************

--> QUESTION NUMBER _ 150 _ about (MC)


QUESTION: "Assessment of a child with rheumatic fever reveals chorea. Which of the following would the nurse consider to be most important?"

CHOICES

( O ) a.) Explain to the child and family that the chorea will disappear over time.

( X ) b.) Institute measures to keep the child in a warm environment.

( X ) c.) Perform neurologic checks every 4 hours until the chorea subsides.

( X ) d.) Promote ambulation by giving aspirin every 4 hours.


RATIONALE: Because the clumsiness and uncontrolled actions can be upsetting to both the child and family, they need to understand that chorea associated with rheumatic fever is not permanent. Measures to keep the child in a warm environment are unnecessary because the child's cardiac workload will increase as the child attempts to remain cool. Neurologic assessments every 4 hours are not necessary because chorea is self-limiting and nonprogressive. Because the child has cardiac involvement, ambulation is contraindicated to minimize the increased oxygen demands on the heart. Aspirin is used primarily as an anti-inflammatory drug and secondarily for pain relief. NURSING PROCESS STEP: Implementation CLIENT NEEDS CATEGORY: Psychosocial integrity CLIENT NEEDS SUBCATEGORY: None
******************************

--> QUESTION NUMBER _ 153 _ about (MC)


QUESTION: "A mother of an ill child is concerned because the child "isn't eating well." Which of the following strategies devised by the mother to help increase the child's intake should the nurse advise against using?"

CHOICES

( X ) a.) Allowing the child to choose his meals from an acceptable list of foods.

( X ) b.) Letting the child substitute items on his tray for other nutritious foods.

( X ) c.) Asking the child to say why he is not eating.

( O ) d.) Telling the child he must eat or else he will not get better.


RATIONALE: Although nutrition plays a large part in the healing process, it is not advisable to tell a child that he will not get better if he does or does not do a particular activity. Not only is this dishonest, it also makes the child believe that his own actions are causing the illness.
Allowing children choices often helps them feel in control. They also will be more likely to eat foods they have chosen. Letting the child substitute items on his tray for other nutritious foods is another way to allow the child to make choices, thus helping him to feel in control. It is important to find out the reason the child is not eating. Clients refuse to eat for multiple reasons, and interventions should be devised taking into consideration the reason for the child's refusal. NURSING PROCESS STEP: Implementation CLIENT NEEDS CATEGORY: Physiological integrity CLIENT NEEDS SUBCATEGORY: Basic care and comfort

******************************

--> QUESTION NUMBER _ 156 _ about (MC)


QUESTION: "When developing a seminar on injury prevention to be presented to a group of parents of children from 2 to 18 years, the nurse would place the first priority on discussing the use of which of the following?"

CHOICES

( O ) a.) Child restraints in automobiles.

( X ) b.) Helmets for biking and skating.

( X ) c.) Special locks for cabinets.

( X ) d.) Topical bug repellent in summer.


RATIONALE: Motor vehicle injuries are the leading cause of death in children older than 1 year of age. Most fatalities are related to nonuse of child restraints and seat belts. Although using helmets for biking and skating safety is important, it is not the priority. Special locks for cabinets are important in the prevention of poisoning, but this is not the priority. Topical bug repellant in summer is important for the prevention of Lyme disease. However, this is not the priority. NURSING PROCESS STEP: Planning CLIENT NEEDS CATEGORY: Safe, effective care environment CLIENT NEEDS SUBCATEGORY: Safety and infection control
******************************

--> QUESTION NUMBER _ 158 _ about (MC)


QUESTION: "The mother of an infant with a congenital heart defect involving decreased pulmonary blood flow tells the nurse that her child has not been gaining weight even with an increased-calorie formula. The mother states that the infant starts out with a good suck but tires and quits after 2 ounces. The infant is receiving oxygen through a nasal cannula as necessary and is on digoxin therapy. Which of the following should the nurse suggest to the mother?"

CHOICES

( X ) a.) Cut a large hole in the nipple.

( X ) b.) Feed the infant every 2 hours.

( X ) c.) Have the infant tested for digoxin toxicity.

( O ) d.) Increase the oxygen for feedings.


RATIONALE: All children use energy to ingest and digest nutrients. The body needs oxygen to use the calories taken in to provide energy. Usually the caloric intake outweighs the energy needed to obtain the nutrients. A child with a congenital heart defect involving decreased pulmonary blood flow that circulates unoxygenated blood to the tissues may need extra oxygen support during times of high energy consumption such as feeding. Without this extra support, the child may become tired. If the child's suck is good, then enlarging the hole in the nipple will give the child too much volume with each suck and may cause the child to choke. Feeding the infant every 2 hours will tire the infant, possibly leading to the ingestion of fewer calories with the next feeding. Tiring during feedings is not a symptom of digoxin toxicity, although lack of appetite may be. NURSING PROCESS STEP: Implementation CLIENT NEEDS CATEGORY: Physiological integrity CLIENT NEEDS SUBCATEGORY: Reduction of risk potential
******************************

--> QUESTION NUMBER _ 160 _ about (MC)


QUESTION: "A young child who has undergone a tonsillectomy refuses to let the nurse look at the tonsillar beds to check for bleeding. To assess whether the child is bleeding from the tonsillar beds, which of the following would be most appropriate?"

CHOICES

( X ) a.) Assess capillary refill.

( X ) b.) Force open the mouth with a tongue blade.

( X ) c.) Monitor for decreased blood pressure.

( O ) d.) Observe for frequent swallowing.


RATIONALE: By observing for frequent swallowing, the nurse can evaluate whether the child is bleeding because blood will go down the back of the throat causing the child to swallow frequently. Decreased peripheral perfusion as evidenced by assessing capillary refill may be a sign of blood loss. In children, however, it is a late sign. Forcing the mouth open with a tongue blade can result in broken teeth, tissue damage, and psychological damage. Although a drop in blood pressure is a sign of blood loss, in children it occurs late. NURSING PROCESS STEP: Assessment CLIENT NEEDS CATEGORY: Physiological integrity CLIENT NEEDS SUBCATEGORY: Reduction of risk potential
******************************

--> QUESTION NUMBER _ 163 _ about (MC)


QUESTION: "When performing a physical assessment on an 18-month-old child, which of the following would be best?"

CHOICES

( O ) a.) Have the mother hold the toddler on her lap.

( X ) b.) Assess the respiratory and cardiac systems first.

( X ) c.) Carry out the assessment from head to toe.

( X ) d.) Assess motor function by having the child run and walk.


RATIONALE: The best strategy for assessing a toddler is to have the parent hold the toddler. Doing so is comforting to the toddler. Assessment should begin with noninvasive assessments first while the child is quiet. Typically these include assessments of the cardiac and respiratory systems. The ears and throat are often examined last. Using a head-to-toe approach is more appropriate for an older child. For a toddler, assessment should begin with noninvasive assessments first while the child is quiet. Having a toddler run and be active may make it difficult to settle the child down after the physical exertion. NURSING PROCESS STEP: Assessment CLIENT NEEDS CATEGORY: Health promotion and maintenance CLIENT NEEDS SUBCATEGORY: None
******************************

--> QUESTION NUMBER _ 169 _ about (MC)


QUESTION: "Which of the following should the nurse do next after noting that an 8-month-old child's posterior fontanel is slightly open?"

CHOICES

( O ) a.) Check the child's head circumference.

( X ) b.) Document this as a normal finding.

( X ) c.) Question the mother about the child's delivery.

( X ) d.) Schedule an x-ray of the child's head.


RATIONALE: The posterior fontanel usually closes by 6 weeks to 2 months. Therefore, the nurse should measure the head circumference to determine if the child's head is larger than the established norms because hydrocephalus can cause separation of the cranium sutures. This is not a normal finding because the posterior fontanel usually closes by age 2 months. Because the child is 8 months old, the delivery history probably would not be a significant factor. An x-ray (radiologic) examination is not necessary until other data are collected. NURSING PROCESS STEP: Implementation CLIENT NEEDS CATEGORY: Health promotion and maintenance CLIENT NEEDS SUBCATEGORY: None
******************************

--> QUESTION NUMBER _ 172 _ about (MC)


QUESTION: "Which of the following interventions would be included in the plan of care for the child with juvenile rheumatoid arthritis to reduce joint pain in the morning just after arising?"

CHOICES

( O ) a.) Having the child sleep in a sleeping bag.

( X ) b.) Increasing pain medication at bedtime.

( X ) c.) Having the child sleep with the joints flexed.

( X ) d.) Awakening the child once nightly to exercise the joints.


RATIONALE: Sleeping in a sleeping bag keeps the joints warm, therefore more flexible. Thus, joint pain in the morning would be lessened. Increasing bedtime pain medications may help the child sleep but will not decrease early morning stiffness. The child's joints should be kept in an extended position during sleep to maintain function. Lack of sleep, such as from awakening the child at night for exercises, is a stressor that can lead to exacerbation of juvenile rheumatoid arthritis. NURSING PROCESS STEP: Planning CLIENT NEEDS CATEGORY: Physiological integrity CLIENT NEEDS SUBCATEGORY: Physiological adaptation
******************************

--> QUESTION NUMBER _ 174 _ about (MC)


QUESTION: "When assessing a 6-month-old child with a large ventricular septal defect, the nurse notices that the child has gained 5 pounds in 1 month. The mother reports that the child has not been wetting many diapers in the last week, although the child is taking the prescribed amounts of formula. "I think it is because he seems to sweat so much." Auscultation of the lung fields reveals fine crackles in the bases. The child's digoxin level is 1 mg/mL. Which of the following nursing diagnoses would be most appropriate?"

CHOICES

( X ) a.) Imbalanced Nutrition: More Than Body Requirements.

( O ) b.) Excess Fluid Volume.

( X ) c.) Risk for Injury.

( X ) d.) Urinary Retention.


RATIONALE: The child is exhibiting characteristics of fluid volume excess related to heart failure. These include decreased output, diaphoresis, weight gain, and crackles. The heart failure is related to left to right shunting that occurs when the child has a large ventral septal defect. No evidence is presented to indicate that altered nutrition is the problem. In fact the mother reports that the child is taking the prescribed amounts of formula. The weight gain is due to the fluid overload. The child's digoxin level is within normal limits. Additionally, there is no evidence to suggest any risk for injury. Although the child's output is decreased, the weight gain is related to fluid overload systemically, not urinary retention. NURSING PROCESS STEP: Analysis, nursing diagnosis CLIENT NEEDS CATEGORY: Physiological integrity CLIENT NEEDS SUBCATEGORY: Physiological adaptation
******************************

--> QUESTION NUMBER _ 176 _ about (MC)


QUESTION: "An 18-month-old child with acquired immunodeficiency syndrome (AIDS) is seen in the clinic for health maintenance. Which of the following vaccines would the nurse anticipate administering to this toddler?"

CHOICES

( O ) a.) Diphtheria-tetanus-acellular pertussis.

( X ) b.) Varicella.

( X ) c.) Measles, mumps, and rubella.

( X ) d.) Hemophilus influenza.


RATIONALE: Diphtheria, acellular pertussis, and tetanus are killed vaccines and may be given to this toddler. Live virus vaccines are not routinely administered to anyone with an altered immune system because multiplication of the virus may be enhanced, causing a severe vaccine-induced illness. Varicella virus vaccine is a live virus vaccine and is not routinely administered to anyone with an altered immune system because multiplication of the virus may be enhanced, causing a severe vaccine-induced illness Measles, mumps, and rubella are live virus vaccines and are not routinely administered to anyone with an altered immune system because multiplication of the virus may be enhanced, causing a severe vaccine-induced illness. Hemophilus influenza vaccine is a live virus vaccine and is not routinely administered to anyone with an altered immune system because multiplication of the virus may be enhanced, causing a severe, vaccine-induced illness NURSING PROCESS STEP: Planning CLIENT NEEDS CATEGORY: Health promotion and maintenance CLIENT NEEDS SUBCATEGORY: None
******************************

--> QUESTION NUMBER _ 178 _ about (MC)


QUESTION: "Parents ask for advice about handling their 2-year-old's negativism. Which of the following would be the best recommendation?"

CHOICES

( X ) a.) Ignore this behavior because it is a stage the child is going through.

( O ) b.) Set realistic limits for the child, then be sure to stick to them.

( X ) c.) Encourage the grandmother to visit frequently to relieve them.

( X ) d.) Punish the child for misbehaving or violating set, strict limits.


RATIONALE: A characteristic of 2-year-olds is negativism, a response to their developing autonomy. Setting realistic limits is important so that the toddler learns what behavior is and is not acceptable. Ignoring the behavior may lead the child to believe that there are no limits. As a result, the child does not learn appropriate behavior. Having the grandmother visit will give the parents a break, but setting limits is more important to the child's development. Limits need to be realistic to ensure that the child learns appropriate behavior. Limits that are too strict are inappropriate, interfering with learning appropriate behavior. NURSING PROCESS STEP: Implementation CLIENT NEEDS CATEGORY: Health promotion and maintenance CLIENT NEEDS SUBCATEGORY: None
******************************

--> QUESTION NUMBER _ 181 _ about (MC)


QUESTION: "When developing a plan of care that includes interventions aimed at preventing complications of a low platelet count in a child with leukemia, which of the following is most appropriate?"

CHOICES

( O ) a.) Consulting with a physician about the use of a stool softener.

( X ) b.) Placing the child in protective isolation.

( X ) c.) Using heparin instead of saline to flush an intermittent IV access device.

( X ) d.) Eliminating raw vegetables and fruits from the child's diet.


RATIONALE: A stool softener would assist in preventing damage to the rectal mucosa due to hard stool, thereby decreasing the chances of rectal bleeding. Placing the child in protective isolation would be appropriate for the child if the neutrophil count was low. The use of heparin is contraindicated in situations in which there is a possibility of increased bleeding due to low platelets. Avoiding raw vegetables or fruits would be indicated if the child's neutrophil count were low. NURSING PROCESS STEP: Planning CLIENT NEEDS CATEGORY: Physiological integrity CLIENT NEEDS SUBCATEGORY: Reduction of risk potential
******************************

--> QUESTION NUMBER _ 185 _ about (MC)


QUESTION: "After resuming feedings in an infant who has undergone a pyloroplasty, which of the following would be most appropriate?"

CHOICES

( X ) a.) Keeping the head of the bed flat with the infant lying supine

( X ) b.) Offering several ounces of an oral electrolyte solution initially.

( X ) c.) Placing the infant in a prone position after each feeding.

( O ) d.) Starting feedings with 5 to 10 mL, slowly increasing amounts as tolerated


RATIONALE: The child who has undergone pyloroplasty often vomits after the first feeding because peristalsis that has been in the right-to-left direction before repair has not reverted to the normal left-to-right direction. Peristalsis reverses as a result of the tightening of the pyloric sphincter, thus not allowing stomach contents to enter the small intestine. Therefore, small feedings of 5 to 10 mL are given and slowly increased as tolerated. Because there is a chance of vomiting, it is not advisable to place an infant supine with the head of the bed flat. If the infant does vomit, aspiration of stomach contents may occur, and pneumonia may result. Small feedings of 5 to 10 mL are given initially and then slowly increased as tolerated. The use of oral electrolyte solutions is unnecessary. The child will have an abdominal incision, so a prone position would be uncomfortable. NURSING PROCESS STEP: Implementation CLIENT NEEDS CATEGORY: Physiological integrity CLIENT NEEDS SUBCATEGORY: Reduction of risk potential
******************************

--> QUESTION NUMBER _ 194 _ about (MC)


QUESTION: "The physician orders eye patching for a child with strabismus. Which of the following statements by the child's mother would indicate the need for additional teaching about this treatment?"

CHOICES

( O ) a.) "You see, his problem eye is patched."

( X ) b.) "I keep the patch on even when he fusses."

( X ) c.) "I have to watch him when he walks because he is clumsy."

( X ) d.) "I take the patch off at night when he goes to bed."


RATIONALE: When an eye patch is used to correct strabismus, the normal eye is patched. That forces the child to use the abnormal, or "lazy," eye, thereby increasing that eye's muscle strength. Keeping the patch on during the child's waking hours, even when he's irritable or fussy, is appropriate to ensure effective treatment. Patching one eye interferes with depth perception and can cause the child to be clumsy at first. The patch can be removed at night while the child sleeps. NURSING PROCESS STEP: Evaluation CLIENT NEEDS CATEGORY: Health promotion and maintenance CLIENT NEEDS SUBCATEGORY: None
******************************

--> QUESTION NUMBER _ 205 _ about (MC)


QUESTION: "A 2-month-old child returns from a cardiac catheterization. The child's fontanel is flat. The diaper is dry. The respiratory rate is 20 breaths/minute and breath sounds are decreased bilaterally. The child is limp although she moves all extremities when stimulated. The dressing over the insertion site is intact, clean, and dry. The pedal pulses are palpable bilaterally and equal to the heart rate. Which of the following nursing diagnoses would be most appropriate?"

CHOICES

( X ) a.) Ineffective Tissue Perfusion related to thrombus formation.

( X ) b.) Deficit Fluid Volume related to inability to take in fluids.

( X ) c.) Risk for Injury related to disruption of vessel integrity.

( O ) d.) Ineffective Breathing Pattern related to sedation.


RATIONALE: The defining characteristics of Ineffective Breathing Pattern include a decrease in respiratory rate and chest expansion, limpness or unresponsiveness, and changes in mental status. In this situation, sedation used during the catheterization is the most probable cause of the Ineffective Breathing Pattern. Because the child's pedal pulses are palpable, Ineffective Tissue Perfusion from a thrombus is unlikely. Because fluids are administered during the procedure, Deficit Fluid Volume is unlikely. A disruption in vessel integrity would lead to bleeding at the site and circulatory or neurologic deficit in the affected leg. The child's dressing is dry and intact and the child is able to move all extremities so Risk for Injury is an inappropriate nursing diagnosis. NURSING PROCESS STEP: Analysis, nursing diagnosis CLIENT NEEDS CATEGORY: Physiological integrity CLIENT NEEDS SUBCATEGORY: Physiological adaptation
******************************

--> QUESTION NUMBER _ 206 _ about (MC)


QUESTION: "A preschool client immobilized in a spica cast complains of having trouble breathing after meals. Which of the following actions would be best? "

CHOICES

( X ) a.) Encourage the client to drink more between meals.

( X ) b.) Teach the child pursed-lip breathing.

( X ) c.) Give the client a laxative after meals.

( O ) d.) Offer the client small feedings several times a day.


RATIONALE: A hip spica cast extends up over the abdomen. Because the abdomen is in a fixed space, abdominal distention secondary to eating pushes the abdominal contents against the diaphragm resulting in decreased chest expansion and subsequent possible respiratory distress. Because the client's complaints are associated with meals, offering small frequent meals provides nutritional support while minimizing distention. Encouraging increased drinking would increase the abdominal distention thus increasing the child's respiratory distress. With a hip spica cast, the child's complaints are due to decreased chest expansion from the abdomen pushing up against the diaphragm. Pursed lip breathing would be effective in preventing air trapping not decreased chest expansion. With a hip spica cast, the child's complaints are due to decreased chest expansion from the abdomen pushing up against the diaphragm. Administering a laxative with meals would be ineffective in relieving the decreased chest expansion. NURSING PROCESS STEP: Implementation CLIENT NEEDS CATEGORY: Physiological integrity CLIENT NEEDS SUBCATEGORY: Basic care and comfort
******************************

--> QUESTION NUMBER _ 212 _ about (MC)


QUESTION: "Which of the following would the nurse include in the plan of care for a child with a fracture in skeletal traction to prevent osteomyelitis?"

CHOICES

( O ) a.) Encouraging the child to eat nutritious foods.

( X ) b.) Administering prophylactic antibiotics as ordered.

( X ) c.) Maintaining the child in reverse isolation.

( X ) d.) Protecting the child from visitors with colds.


RATIONALE: The best prevention strategy for osteomyelitis, a bacterial infection of the bone, is to maintain skin integrity and promote good nutrition. Encouraging the intake of nutritional foods is essential to ensure bone repair and healing, thereby minimizing the risk of infection. Unless the child already has a bacterial infection, antibiotics are not administered prophylactically when skeletal traction is used. Maintaining reverse isolation is not necessary for this child and could lead to social isolation. Protecting the child from visitors with colds is inappropriate because colds are caused by viruses while osteomyelitis is caused by bacteria invading bone tissue. Additionally, restricting visitors could lead to social isolation. NURSING PROCESS STEP: Planning CLIENT NEEDS CATEGORY: Physiological integrity CLIENT NEEDS SUBCATEGORY: Reduction of risk potential
******************************

--> QUESTION NUMBER _ 214 _ about (MC)


QUESTION: "Which of the following would the nurse most likely assess in a child who has sustained full-thickness burns?"

CHOICES

( X ) a.) Blanching to the touch.

( X ) b.) Excessive bleeding.

( O ) c.) Minimal complaints of pain.

( X ) d.) Blistering, moist appearance.


RATIONALE: Full-thickness burns are serious injuries in which all the skin layers are destroyed. Lack of pain is characteristic of full-thickness burns. With full-thickness burns, blanching and bleeding are absent because blood supply is destroyed. With full-thickness burns, blanching and bleeding are absent because blood supply is destroyed. Blisters and moist appearance characterize partial-thickness burns. NURSING PROCESS STEP: Assessment CLIENT NEEDS CATEGORY: Physiological integrity CLIENT NEEDS SUBCATEGORY: Physiological adaptation
******************************

--> QUESTION NUMBER _ 216 _ about (MC)


QUESTION: "After teaching the parents of a 15-month-old child who has undergone cleft palate repair how to use elbow restraints, which of the following statements by the parents indicates effective teaching?"

CHOICES

( X ) a.) "We'll keep the restraints in place continuously until the doctor says it's okay to remove them."

( X ) b.) "We can take off the restraints while our child is playing but we'll make sure to put them back on at night."

( X ) c.) "The restraints should be taped directly to our child's arms so that they will stay in one place."

( O ) d.) "We'll remove the restraints temporarily at least three times a day to check his skin then put them right back on."


RATIONALE: Elbow restraints help to keep the child from placing fingers or any other object in the mouth that would cause injury to the operative site. The restraints are worn at all times except when they are removed to check the skin. Because of the risk for skin breakdown, the restraints are removed periodically during the day to assess the child's underlying skin. It is advisable to remove only one restraint at a time while keeping hold of the child's hand on the unrestrained side. Toddlers are quick and usually want to explore the area in the mouth that the surgery has made feel different. The restraints should be in place at all times during sleep and play to prevent inadvertent injury to the operative site. Toddlers are quick and usually want to explore the area in the mouth that the surgery has made feel different. Taping the restraints directly to the skin is not advised because skin breakdown can occur when tape is reapplied to the same area over several weeks. The restraints can be fastened to clothing to keep them from slipping. NURSING PROCESS STEP: Evaluation CLIENT NEEDS CATEGORY: Safe, effective care environment CLIENT NEEDS SUBCATEGORY: Safety and infection control
******************************

--> QUESTION NUMBER _ 219 _ about (MC)


QUESTION: "Immediately after the return of an 18-month-old child to his room following insertion of a ventriculoperitoneal shunt, which of the following would the nurse do first?"

CHOICES

( X ) a.) Ask the child to state his name and where he is.

( X ) b.) Palpate his anterior fontanel.

( O ) c.) Position him on the side opposite the shunt site.

( X ) d.) Check his pupil size and reactivity to light.


RATIONALE: As soon as the child returns to his room, he needs to be positioned appropriately, in this case on the side opposite the shunt placement to avoid pressure on the operative site. Developmentally, the child at this age may or may not be able to state his name or where he is. Palpating his fontanel and checking pupils are part of the neurologic assessment that would be done once the child is positioned properly. Checking the child's pupils is part of the neurologic assessment that would be done once the child is positioned properly. NURSING PROCESS STEP: Implementation CLIENT NEEDS CATEGORY: Physiological integrity CLIENT NEEDS SUBCATEGORY: Reduction of risk potential
******************************

--> QUESTION NUMBER _ 220 _ about (MC)


QUESTION: "After uncomplicated abdominal surgery, which of the following would be most appropriate when determining if an alert school-aged child is ready to drink oral fluids? "

CHOICES

( X ) a.) Ask if the child wants something to drink.

( O ) b.) Auscultate the child's abdomen for bowel sounds.

( X ) c.) Determine that the child has a gag reflex.

( X ) d.) Palpate the epigastric area for discomfort.


RATIONALE: After uncomplicated abdominal surgery, fluid intake is resumed early in the postoperative period. However, before fluids are given, the nurse needs to auscultate the child's abdomen for bowel sounds indicating the return of peristalsis and a functioning gastrointestinal tract. Fluids are withheld until bowel sounds are heard. Asking the child if he or she wants something to drink is inappropriate because medications used before and during surgery may cause thirst. Additionally, the child's degree of thirst is not an indicator for peristalsis. Determining if a gag reflex is present would be more appropriate for the child having undergone upper gastrointestinal procedures such as gastroscopy. Having a gag reflex is usually not a concern in a child who is alert and has had uncomplicated abdominal surgery. Palpating the epigastric area or abdomen for discomfort provides no information about the function of the gastrointestinal tract. Complaints of pain are likely because the client has had abdominal surgery. NURSING PROCESS STEP: Assessment CLIENT NEEDS CATEGORY: Physiological integrity CLIENT NEEDS SUBCATEGORY: Physiological adaptation
******************************

--> QUESTION NUMBER _ 221 _ about (MC)


QUESTION: "The mother of a 15-month-old child who is coughing and having trouble breathing telephones the clinic to ask advice because she suspects that her child has croup. Which of the following instructions would be most appropriate?"

CHOICES

( X ) a.) Administer acetaminophen (Tylenol) every 4 hours.

( O ) b.) Take the child into the bathroom and run the hot water.

( X ) c.) Give over-the-counter cough syrup every 6 hours.

( X ) d.) Get the child to take as much fluid as possible.


RATIONALE: For the child with croup who is coughing and having difficulty breathing, the child should be taken into the shower where hot water is running to make the bathroom steamy. Steam helps to loosen secretion and relieve some of the respiratory distress. Giving acetaminophen is helpful but will not ease difficult breathing. Giving over-the-counter cough syrup is inappropriate because the underlying problem is airway inflammation and subsequent mucus accumulation and bronchoconstriction. Getting the child to take as much fluid as possible is important but it will not be effective in easing difficult breathing. NURSING PROCESS STEP: Implementation CLIENT NEEDS CATEGORY: Physiological integrity CLIENT NEEDS SUBCATEGORY: Physiological adaptation
******************************

--> QUESTION NUMBER _ 227 _ about (MC)


QUESTION: "When planning a screening clinic for scoliosis, the nurse would anticipate targeting which of the following groups?"

CHOICES

( O ) a.) Preadolescents at the beginning of a growth spurt.

( X ) b.) Toddlers who have diets low in calcium and vitamin D.

( X ) c.) Preschoolers who are entering kindergarten.

( X ) d.) Infants whose mothers have had no prenatal care.


RATIONALE: Preadolescents are at greatest risk for scoliosis because of the growth associated with this age group. Incidence is higher in girls than boys and increases during periods of rapid growth. A toddler with a diet low in vitamin D and calcium is prone to develop rickets. The risk for scoliosis is greatest during adolescence, not for preschoolers. However, prior to entering school, preschoolers are required to have their immunizations up-to-date. No relationship exists between poor prenatal care and scoliosis. NURSING PROCESS STEP: Planning CLIENT NEEDS CATEGORY: Health promotion and maintenance CLIENT NEEDS SUBCATEGORY: None
******************************

--> QUESTION NUMBER _ 229 _ about (MC)


QUESTION: "A 23-month-old child pulled a pan of hot water off the stove and spilled it onto her chest and arms. Her mother was right there when it happened. Which of the following should the mother have done immediately?"

CHOICES

( X ) a.) Apply ice directly to the burned areas.

( O ) b.) Place the child in the bathtub of cool water.

( X ) c.) Apply antibiotic ointment to the burned areas.

( X ) d.) Call the neighbor to come over and help her.


RATIONALE: The emergency treatment of both minor and major burns includes stopping the burning process by immersing the burned area in cool, but not cold, water. Thus, the mother should place the child in a bathtub of cool water. Applying ice directly to the burned area is inappropriate at this time because more tissue damage can result. Antibiotic ointment should not be applied to the burned area at this time because the burning process must be stopped first. Calling a neighbor for help is appropriate after she has placed then removed her child from the bathtub. NURSING PROCESS STEP: Implementation CLIENT NEEDS CATEGORY: Physiological integrity CLIENT NEEDS SUBCATEGORY: Reduction of risk potential
******************************

--> QUESTION NUMBER _ 231 _ about (MC)


QUESTION: "When completing an assessment of a healthy adolescent client, which of the following would be most appropriate?"

CHOICES

( X ) a.) Obtain a detailed account of the adolescent's prenatal and early developmental history.

( X ) b.) Discuss sexual preferences and behaviors with the parents present for legal reasons.

( X ) c.) Discuss the client's smoking with parents present in the room.

( O ) d.) Assess the adolescent in private; gather additional information from the parents.


RATIONALE: When assessing an adolescent, it is appropriate to first obtain information from the adolescent in private then interview the parents for additional information. Doing so helps to promote independence and responsibility for self-care. Obtaining prenatal and early developmental history information is usually not important for a healthy adolescent. In addition, this information typically would have already been obtained at an earlier age. No legal reason would prohibit the nurse from discussing sexuality with the adolescent without the parents present. Discussing smoking with the parents present in the room is inappropriate. If the adolescent smokes, the parents may be unaware and the adolescent would lose trust in the nurse. When assessing an adolescent, it is appropriate to first obtain information from the adolescent in private then interview the parents for additional information. Doing so helps to promote independence and responsibility for self care. NURSING PROCESS STEP: Implementation CLIENT NEEDS CATEGORY: Health promotion and maintenance CLIENT NEEDS SUBCATEGORY: None
******************************

--> QUESTION NUMBER _ 233 _ about (MC)


QUESTION: "At 3 AM, the mother of a 3-year-old child calls the emergency room nurse and reports the child has a temperature of 101.1%F (38.4%C), a runny nose, and a barky cough that "gets going and won't stop." The mother states that she just gave the child acetaminophen (Tylenol). Which of the following should the nurse recommend next?"

CHOICES

( O ) a.) Sitting with the child in a steamy warm bathroom.

( X ) b.) Running a steam vaporizer near the child's bedside.

( X ) c.) Giving the child an over-the-counter decongestant.

( X ) d.) Administering aspirin in 2 hours.


RATIONALE: Based on the mother's description, the child most likely is exhibiting signs and symptoms of laryngotracheal bronchitis. The mother should try to decrease the inflammation in the upper airway by exposing her child to a warm, steamy environment. The safest method is to steam up the bathroom and stay with the child. Steam vaporizers work by boiling water. Their use is to be avoided because they can cause severe burns if the child comes in close contact with the steam or if the vaporizer spills. A decongestant may assist in decreasing the rhinorrhea (runny nose) but it will not decrease the inflammation in the upper airway. Laryngotracheal bronchitis is caused by a virus. Aspirin is contraindicated in children with viral infections because this combination is implicated in Reye's syndrome. NURSING PROCESS STEP: Implementation CLIENT NEEDS CATEGORY: Physiological integrity CLIENT NEEDS SUBCATEGORY: Physiological adaptation
******************************

--> QUESTION NUMBER _ 235 _ about (MC)


QUESTION: "When preparing to conduct prenatal and parenting classes for a group of parents, the clinic's nursing staff will be providing childcare for the parents' children who range in age from 13 months to 6 years. The clinic has a playroom. Which of the following activities would be most appropriate to include?"

CHOICES

( O ) a.) Free play with adult supervision.

( X ) b.) A group sing-along.

( X ) c.) Drawing and painting projects.

( X ) d.) Viewing cartoon videos.


RATIONALE: Planning any single activity that will appeal to children from ages 13 months to 6 years is next to impossible because of the developmental differences found in such a wide age group. It would be best to allow these children to participate in free play with adult supervision. A group sing-along would be appropriate for preschoolers and school-aged children. However, toddlers have short attention spans and would most likely find it difficult to participate in a group activity, such as a sing-along, for long. Although drawing and painting projects would be appropriate for preschoolers and school-aged children, toddlers have a tendency to put objects into their mouths. Therefore, drawing and painting projects would be inappropriate for this age group. Viewing cartoon videos would be inappropriate for young toddlers, who typically have short attention spans. Additionally, young toddlers may not understand the videos. NURSING PROCESS STEP: Planning CLIENT NEEDS CATEGORY: Health promotion and maintenance CLIENT NEEDS SUBCATEGORY: None
******************************

--> QUESTION NUMBER _ 236 _ about (MC)


QUESTION: "While examining a 12-month-old child, the nurse notes that the child can stand independently but cannot walk without support. Which of the following actions would be most appropriate?"

CHOICES

( X ) a.) Ask the mother if the child uses a walker at home.

( O ) b.) Do nothing because this is a normal finding in a child this age.

( X ) c.) Initiate a consultation with a developmental specialist.

( X ) d.) Tell the mother that the child may have a developmental delay.


RATIONALE: A child aged 12 months is expected to cruise but not necessarily walk without support. Using or not using a walker at home does not significantly affect independent walking. A developmental specialist consult is not necessary. Even if the child's development in walking is slow, this fact is not sufficient data to make the nurse suspect developmental delay. Even if the child's development in walking is slow, this fact is not sufficient data to make the nurse suspect developmental delay. NURSING PROCESS STEP: Implementation CLIENT NEEDS CATEGORY: Health promotion and maintenance CLIENT NEEDS SUBCATEGORY: None
******************************

--> QUESTION NUMBER _ 240 _ about (MC)


QUESTION: "Which of the following nursing diagnoses would the nurse identify as the priority for a 4-month-old infant with heart failure and congenital heart disease?"

CHOICES

( O ) a.) Activity Intolerance.

( X ) b.) Risk for Infection.

( X ) c.) Impaired Mobility.

( X ) d.) Ineffective Health Maintenance.


RATIONALE: An infant with congenital heart disease and congestive heart failure usually tires easily, leading to a priority nursing diagnosis of Activity Intolerance. Nursing care needs to focus on allowing the infant to have frequent rest periods. Infants with congenital heart disease and congestive heart failure are not necessarily at risk for more infections than other infants. Impaired Mobility usually is not a problem because an infant with congenital heart disease usually exhibits normal physical mobility. Ineffective Health Maintenance usually is not a problem because these infants still need regular and routine health check-ups. NURSING PROCESS STEP: Analysis, nursing diagnosis CLIENT NEEDS CATEGORY: Physiological integrity CLIENT NEEDS SUBCATEGORY: Physiological adaptation
******************************

--> QUESTION NUMBER _ 243 _ about (MC)


QUESTION: "After the nurse has taught the parents of a 5-year-old boy who has leukemia how to talk with their child about death and dying, which of the following would indicate that the parents have age-appropriate expectations about their child's reaction to his impending death?"

CHOICES

( X ) a.) "He is too young to understand what is happening to him."

( O ) b.) "He might think he can cause his death because he has misbehaved."

( X ) c.) "He will accept his death as caused by his disease."

( X ) d.) "He will understand how much his siblings will miss him."


RATIONALE: A 5-year-old child is in the preoperational stage of cognitive development and thinks of death as temporary. Also, for a child this age, thinking about behavior often is believed to be magical; thus, the child may think that his behavior can cause death. Generally, children under 3 years of age are unable to differentiate death from temporary separation and are unable to understand what is happening. Logical thinking, evidenced by accepting death due to his disease, would occur during Piaget's stage of concrete operations between ages 6 and 12 years. Although a 5-year-old child will be able to understand that he will be missed, he lacks the cognitive development to understand the extent of how much his siblings will miss him. NURSING PROCESS STEP: Evaluation CLIENT NEEDS CATEGORY: Health promotion and maintenance CLIENT NEEDS SUBCATEGORY: None
******************************

--> QUESTION NUMBER _ 246 _ about (MC)


QUESTION: "Initial nursing interventions for a child admitted to the hospital with a diagnosis of meningitis due to H. influenzae should include which of the following?"

CHOICES

( X ) a.) Keeping the child well hydrated.

( O ) b.) Maintaining a quiet, cool environment.

( X ) c.) Keeping the child positioned flat in the bed.

( X ) d.) Placing the child on airborne precautions.


RATIONALE: The child with meningitis should be kept in a quiet, cool environment to minimize stimulation, thus helping to decrease intracranial pressure. The child's hydration status requires a careful balance. Any fluid deficit should be corrected. Then the child should be kept on low fluid maintenance to prevent cerebral edema. To decrease intracranial pressure and facilitate venous return, the child should be positioned with the head of the bed elevated and the head midline. A child with meningitis does not need to be placed on airborne precautions. Rather, the treatment is droplet precautions because meningitis caused by H. influenzae is transmitted via contact with the conjunctivae or mucous membranes of the nose or mouth of a susceptible person via sneezing, coughing, or talking. NURSING PROCESS STEP: Implementation CLIENT NEEDS CATEGORY: Physiological integrity CLIENT NEEDS SUBCATEGORY: Physiological adaptation
******************************

--> QUESTION NUMBER _ 249 _ about (MC)


QUESTION: "A mother asks the nurse when she should wean her 4-month-old infant from breast-feeding and begin using a cup. Which of the following would the nurse explain as the best indication of the infant's readiness to be weaned?"

CHOICES

( X ) a.) Taking solid foods well.

( X ) b.) Sleeping through the night.

( O ) c.) Shortening the nursing time.

( X ) d.) Eating on a regular schedule.


RATIONALE: Readiness for weaning is an individual matter but is usually indicated when an infant begins to decrease the time spent nursing. The infant is then showing independence and will soon be ready to take a cup and learn a new skill. The infant ready for weaning may also demonstrate an ability to take solid foods well, sleep through the night, and eat on a regular schedule. These behaviors though are not necessarily the best evidence of readiness for weaning. The infant ready for weaning may also demonstrate an ability to take solid foods well, sleep through the night, and eat on a regular schedule. These behaviors though are not necessarily the best evidence of readiness for weaning. The infant ready for weaning may also demonstrate an ability to take solid foods well, sleep through the night, and eat on a regular schedule. These behaviors though are not necessarily the best evidence of readiness for weaning. NURSING PROCESS STEP: Implementation CLIENT NEEDS CATEGORY: Health promotion and maintenance CLIENT NEEDS SUBCATEGORY: None
******************************

--> QUESTION NUMBER _ 250 _ about (MC)


QUESTION: "The mother says that the infant's physician recommends certain foods, but the infant refuses to eat them after breast-feeding. The nurse should suggest that the mother alter the feeding plan by doing which of the following?"

CHOICES

( X ) a.) Offering dessert followed by some vegetables and meat.

( X ) b.) Offering breast milk as long as the infant refuses to eat solid foods.

( X ) c.) Mixing pureed food with some breast milk in a bottle with a large-holed nipple.

( O ) d.) Allowing the infant to nurse for a few minutes then offering solid foods.


RATIONALE: It is typical for an infant just starting on solid foods to spit them out because the infant does not know how to swallow them. Also, the infant is hungry and is accustomed to having milk to satisfy that hunger. It is generally recommended that an infant be given some milk first then offered solid foods. Offering dessert followed by vegetables and meat is inappropriate because the infant will learn to prefer the sweets first, possibly refusing intake of the vegetables and meats. Offering breast milk as long as the infant refuses solid foods is inappropriate because an infant who takes all the milk first will have no interest in the solids. Mixing pureed foods with cow's or breast milk is inappropriate because solid food should be given by a spoon. Also, using a large-holed nipple may cause the infant to choke from getting too much fluid at one time. NURSING PROCESS STEP: Implementation CLIENT NEEDS CATEGORY: Health promotion and maintenance CLIENT NEEDS SUBCATEGORY: None
******************************

--> QUESTION NUMBER _ 251 _ about (MC)


QUESTION: "After the nurse instructs the parents of a 5-month-old infant about the purpose of the Denver Developmental Screening Test (DDST), which of the following statements by the parents about what the test measures would indicate that the teaching was effective?"

CHOICES

( X ) a.) Intelligence quotient.

( X ) b.) Emotional development.

( O ) c.) Social and physical abilities.

( X ) d.) Potential for future development.


RATIONALE: The Denver Developmental Screening Test (DDST) measures a child's social, language, and fine and gross motor skills by testing abilities that usually occur at a given age. The DDST is not designed to measure intelligence or emotional development nor does it necessarily predict future development. The DDST is not designed to measure intelligence or emotional development nor does it necessarily predict future development. The DDST is not designed to measure intelligence or emotional development nor does it necessarily predict future development. NURSING PROCESS STEP: Evaluation CLIENT NEEDS CATEGORY: Health promotion and maintenance CLIENT NEEDS SUBCATEGORY: None
******************************

--> QUESTION NUMBER _ 252 _ about (MC)


QUESTION: "When discussing a 7-month-old infant's motor skill development with the mother, the nurse should explain that by age 7 months, an infant most likely will be able to do which of the following?"

CHOICES

( X ) a.) Walk with one hand held.

( X ) b.) Eat successfully with a spoon.

( X ) c.) Stand while holding onto furniture.

( O ) d.) Sit alone using the hands for support.


RATIONALE: By age 6 months, an infant can sit alone, leaning forward on the hands for support. The ability to sit follows progressive head control and straightening of the back. By 12 months, an infant can walk with one hand held. At about 18 months, an infant can eat successfully with a spoon. At 11 months, an infant can stand and walk while holding onto furniture. NURSING PROCESS STEP: Implementation CLIENT NEEDS CATEGORY: Health promotion and maintenance CLIENT NEEDS SUBCATEGORY: None
******************************

--> QUESTION NUMBER _ 253 _ about (MC)


QUESTION: "The nurse teaches the parent about the normal reaction that an infant may experience 12 to 24 hours after DTaP immunization. The nurse determines that the teaching is effective when the parent asks which of the following?"

CHOICES

( X ) a.) "Will the lethargy make it harder to breast-feed?"


( O ) b.) "How much acetaminophen (Tylenol) can I give for the fever?"


( X ) c.) "Can you give loperamide (Imodium) to an infant?"


( X ) d.) "What kind of nose spray can I use for the baby's congestion?"



RATIONALE: Mild fever is common in an infant at 12 to 24 hours after administration of a DTaP vaccine. The mother should be taught to give the infant acetaminophen for the fever. Temperature above 102%F (38.9%C) (measured rectally) should be reported to the physician. After DTaP immunization, a mild fever is common. Typically an infant with a fever is restless rather than lethargic. Diarrhea (for which loperamide [Imodium] is given in adults, not infants) is not associated with administration of the DTaP vaccine. Nasal congestion is not associated with the DTaP vaccine. NURSING PROCESS STEP: Evaluation CLIENT NEEDS CATEGORY: Physiological integrity CLIENT NEEDS SUBCATEGORY: Pharmacological and parenteral therapies
******************************

--> QUESTION NUMBER _ 254 _ about (MC)


QUESTION: "On observing a parent propping a bottle for a 2-month-old child in the waiting room, the nurse explains the dangers of this to the parent. Which of the following statements indicates that the parent has understood the nurse's teaching?"

CHOICES

( X ) a.) "I didn't know it would cause my baby to gain too much weight."


( O ) b.) "I can see how it might cause choking, but how does it cause dental caries?"


( X ) c.) "So, because I prop the bottle, I might have trouble weaning the child?"


( X ) d.) "I will stop propping the bottle so my child will sleep through the night."



RATIONALE: Many mothers prop a bottle of formula or fruit juice for their infants at bedtime. The infant then awakens periodically to take more formula or juice, constantly bathing the teeth with high-carbohydrate liquid that predisposes the infant to dental caries. Choking is also a risk because of the fluids dripping from the hole in the nipple if the child falls asleep while the nipple is still in the mouth. Propping a bottle does not necessarily lead to obesity, abnormally prolonged use of a bottle, or nighttime feedings. Propping a bottle does not necessarily lead to obesity, abnormally prolonged use of a bottle, or nighttime feedings. Propping a bottle does not necessarily lead to obesity, abnormally prolonged use of a bottle, or nighttime feedings. NURSING PROCESS STEP: Evaluation CLIENT NEEDS CATEGORY: Physiological integrity CLIENT NEEDS SUBCATEGORY: Reduction of risk potential
******************************

--> QUESTION NUMBER _ 255 _ about (MC)


QUESTION: "The mother of a 9-month-old infant asks about adding new foods to his diet. The child is being breast-fed and takes formula and cereal when at the sitter's. Which of the following would the nurse instruct the mother to do?"

CHOICES

( X ) a.) Mix new foods with formula or breast milk.

( X ) b.) Mix new foods with more familiar foods.

( O ) c.) Offer new foods one at a time.

( X ) d.) Offer new foods after giving formula or breast milk.


RATIONALE: Infants should be offered new foods one at a time. This gives the infant the chance to become gradually familiar with a variety of food tastes and textures and also helps identify any allergies or adverse reactions to a specific food. Mixing new foods with formula, breast milk, or other familiar foods would make it impossible to satisfactorily detect allergic or other unfavorable reactions. Mixing new foods with formula, breast milk, or other familiar foods would make it impossible to satisfactorily detect allergic or other unfavorable reactions. This practice may also cause the infant to refuse familiar foods. If a new food is offered after the infant's appetite is satisfied with formula or breast milk, the infant is not likely to eat the new food. NURSING PROCESS STEP: Implementation CLIENT NEEDS CATEGORY: Health promotion and maintenance CLIENT NEEDS SUBCATEGORY: None
******************************

--> QUESTION NUMBER _ 256 _ about (MC)


QUESTION: "A mother brings her 18-month-old child to the clinic because the child "eats ashes, crayons, and paper." Which of the following information would be most important to obtain first about this toddler?"

CHOICES

( X ) a.) Currently cutting large teeth.

( X ) b.) Experiencing a growth spurt.

( O ) c.) Experiencing changes in the home environment.

( X ) d.) Eating a soft, low-roughage diet.


RATIONALE: It is important to determine if the child is experiencing any change in the home environment that could cause anxiety that is relieved through oral gratification. A craving to eat nonfood substances is known as pica. Nutritional deficiencies, especially iron deficiency, were once thought to cause pica but research has not substantiated this theory. The child is demonstrating a craving to eat nonfood substances, known as pica. The cutting of large teeth is an unlikely cause of pica. The child is demonstrating a craving to eat nonfood substances, known as pica. Growth spurts are considered to be an unlikely cause of pica. The child is demonstrating a craving to eat nonfood substances, known as pica. Eating a low-roughage diet is considered to be an unlikely cause of pica. NURSING PROCESS STEP: Assessment CLIENT NEEDS CATEGORY: Health promotion and maintenance CLIENT NEEDS SUBCATEGORY: None
******************************

--> QUESTION NUMBER _ 257 _ about (MC)


QUESTION: "Which of the following behaviors absent in an 18-month-old child would cause the nurse to be concerned?"

CHOICES

( X ) a.) Copying a circle.

( O ) b.) Playing with pull toys.

( X ) c.) Playing tag with other children.

( X ) d.) Building a tower of eight blocks.


RATIONALE: Playing with pull toys is a typical task of a normally developed 18-month-old child. Inability of the toddler to do so would be a concern. Copying a circle is a behavior typical of a 3-year-old child. Playing tag with other children requires cooperative play and the ability to follow rules; this behavior develops at about age 5 years. Building a tower of eight or more blocks is typical behavior of a 3-year-old child. NURSING PROCESS STEP: Analysis CLIENT NEEDS CATEGORY: Health promotion and maintenance CLIENT NEEDS SUBCATEGORY: None
******************************

--> QUESTION NUMBER _ 258 _ about (MC)


QUESTION: "Which of the following nursing diagnoses would be most appropriate when teaching the mother of a toddler?"

CHOICES

( X ) a.) Activity Intolerance.

( O ) b.) Risk for Injury.

( X ) c.) Delayed Growth and Development.

( X ) d.) Impaired Mobility.


RATIONALE: The most appropriate nursing diagnosis would be Risk for Injury because a toddler is typically engaged in exploring the environment while becoming increasing mobile. Safety issues are an important part of anticipatory guidance with parents of toddlers. Normally, a toddler would be very active. Thus, Activity Intolerance would be inappropriate. Normally, a toddler would not have Delayed Growth and Development unless there was an underlying problem. Normally a toddler would not have Impaired Mobility unless the child had an underlying problem or condition. NURSING PROCESS STEP: Analysis, nursing diagnosis CLIENT NEEDS CATEGORY: Health promotion and maintenance CLIENT NEEDS SUBCATEGORY: None
******************************

--> QUESTION NUMBER _ 259 _ about (MC)


QUESTION: "During a routine health assessment, a mother tells the nurse that her 2-year-old child is using a potty seat but is still having problems toilet-training. Which of the following suggestions would be most appropriate?"

CHOICES

( X ) a.) Offer the child more praise each time.

( X ) b.) Use a potty chair instead of a potty seat.

( X ) c.) Focus on the "accidents" that occur during training.

( O ) d.) Defer training until the child is developmentally ready.


RATIONALE: The most common reason for failed toilet- training is that the child is simply not developmentally ready for training. Even with appropriate rewards and proper equipment, the child who is not ready for training will not be able to learn voluntary control. Offering praise is important but only effective when the child is developmentally ready. Using a potty chair may help but success occurs only when the child is developmentally ready. "Accidents" during training should be ignored. They are usually caused by the child's incomplete sphincter control and to poor recognition of the impending need to defecate until it is too late to get to the potty chair. NURSING PROCESS STEP: Implementation CLIENT NEEDS CATEGORY: Health promotion and maintenance CLIENT NEEDS SUBCATEGORY: None
******************************

--> QUESTION NUMBER _ 260 _ about (MC)


QUESTION: "A 2 1/2-year-old child and his 2-month-old sibling are brought to the clinic by their father, who explains that the older child says "no" whenever asked to do something. The nurse would explain that the negativism demonstrated by toddlers is frequently an expression of which of the following?"

CHOICES

( O ) a.) Pursuit of autonomy.

( X ) b.) Need to expend excess energy.

( X ) c.) Separation anxiety.

( X ) d.) Sibling rivalry.


RATIONALE: According to Erikson, the developmental task of toddlerhood is acquiring a sense of autonomy while overcoming a sense of doubt and shame. Characteristics of negativism and ritualism are typical behaviors in this quest for autonomy. The toddler often does the opposite of what others request. Hyperactivity or the need to expend excess energy is a typical behavior that may be demonstrated by a toddler. However, this behavior is the not the basis for the toddler's negativism. Separation anxiety is a behavior that may be demonstrated by the toddler. However, this behavior is the not the basis for the toddler's negativism. Sibling rivalry is a behavior that may be demonstrated by the toddler but it does not explain a toddler's negativism. NURSING PROCESS STEP: Implementation CLIENT NEEDS CATEGORY: Health promotion and maintenance CLIENT NEEDS SUBCATEGORY: None
******************************

--> QUESTION NUMBER _ 261 _ about (MC)


QUESTION: "The father of a 3-year-old child expresses a concern about the child's fear of the dark. Using Piaget's concepts of cognitive development, which of the following would be the most appropriate explanation as the basis for the child's fear of darkness?"

CHOICES

( X ) a.) Reversibility.

( O ) b.) Animism.

( X ) c.) Conservation of matter.

( X ) d.) Object permanence.


RATIONALE: Animism (the child attributes the quality of conscious thought to inanimate objects) is a peculiarity of preconceptual thought. According to Piaget, the preconceptual phase of cognitive development, part of the preoperational stage, lasts from age 2 to 4 years. The concept of reversibility (an act can be undone by performing an opposite act) is characteristic of children in Piaget's concrete operational stage (school age). Reversibility allows mental action to replace physical action. The concept of conservation (things remain the same even when their form and shape change) is characteristic of the school-aged child. Object permanence, a milestone of the sensorimotor period of Piaget's theory, is demonstrated at age 6 to 9 months when the infant reaches for a hidden object. NURSING PROCESS STEP: Implementation CLIENT NEEDS CATEGORY: Health promotion and maintenance CLIENT NEEDS SUBCATEGORY: None
******************************

--> QUESTION NUMBER _ 262 _ about (MC)


QUESTION: "A parent reports that his 2-year-old child often falls when running. The nurse interprets this as indicating which of the following as a normal aspect of a toddler's vision?"

CHOICES

( O ) a.) Near-sightedness.

( X ) b.) Far-sightedness.

( X ) c.) Binocular vision.

( X ) d.) Strabismus.


RATIONALE: Until age 7 years, children are normally myopic and nearsighted. Additionally toddlers lack motor coordination and their depth perception is not well developed, placing them at risk for falling. Until age 7 years, children are normally myopic and nearsighted. Hyperopia, or farsightedness, is the ability to see objects at a distance but not at close range. Binocular vision (seeing with both eyes in stereo) assists in depth perception. Thus, the child would be less clumsy. However, depth perception in a toddler is not well developed. Strabismus is misalignment of the eyes and can result from muscle weakness or trauma. It is not a normal finding in a toddler. NURSING PROCESS STEP: Analysis CLIENT NEEDS CATEGORY: Health promotion and maintenance CLIENT NEEDS SUBCATEGORY: None
******************************

--> QUESTION NUMBER _ 263 _ about (MC)


QUESTION: "A mother brings her 2-year-old child to the clinic because of her concerns about the child's nutritional status. "For the last week, he has refused to eat anything except animal crackers and peanut butter and jelly sandwiches." Which of the following measures would be most appropriate for the nurse to suggest?"

CHOICES

( X ) a.) Giving the child a small reward if he eats extra food given to him.

( X ) b.) Consulting a physician because this behavior could lead to a nutritional deficiency.


( O ) c.) Not worrying about this behavior because food fads usually last only a short time.

( X ) d.) Insisting the child eat small portions of the family's meal to maintain adequate nutrition.


RATIONALE: During the toddler years, food preferences and appetite are changeable. A child may enjoy one food for several days in a row then suddenly refuse to eat it again for days. Attempts to alter such food fads are met with resentment and obstinacy. Accepting such extremes and offering small portions of other foods is most appropriate. Offering rewards is not an appropriate nutritional strategy because eating then becomes a power struggle between the child and parent. Consulting a physician is unnecessary because food fads are normal and usually temporary. Insisting that the child eat small portions of the family's meal is not an appropriate nutritional strategy. Insisting on how or what the toddler eats sets up the potential for a power struggle between the child and parent that focuses on eating. NURSING PROCESS STEP: Implementation CLIENT NEEDS CATEGORY: Health promotion and maintenance CLIENT NEEDS SUBCATEGORY: None
******************************

--> QUESTION NUMBER _ 264 _ about (MC)


QUESTION: "After the nurse assesses a 2 1/2-year-old's teeth during the physical examination, which of the following instructions would the nurse give to the mother?"

CHOICES

( X ) a.) Make sure the child brushes his teeth after every meal and at bedtime.

( X ) b.) Give the child a small, soft-bristle toothbrush to use.

( O ) c.) Floss the child's teeth using dental floss.

( X ) d.) Add a fluoride supplement to the child's milk.


RATIONALE: For a toddler, a parent should clean and floss the toddler's teeth because the child does not have the cognitive or motor skills needed for effective cleaning. The child lacks the cognitive or motor skills needed for effective cleaning. Therefore, rather than making sure the child brushes his teeth, the parent should brush the toddler's teeth after every meal and at bedtime using a small toothbrush with soft, rounded nylon bristles that are short and uniform in length. Although a small soft-bristle toothbrush is appropriate, the child lacks the cognitive or motor skills needed for effective cleaning. Therefore, the parent should brush the toddler's teeth after every meal and at bedtime using a small toothbrush with soft, rounded nylon bristles that are short and uniform in length. A fluoride supplement is needed only if the child ingests minimal amounts of tap water or the family has well water. NURSING PROCESS STEP: Implementation CLIENT NEEDS CATEGORY: Health promotion and maintenance CLIENT NEEDS SUBCATEGORY: None
******************************

--> QUESTION NUMBER _ 265 _ about (MC)


QUESTION: "Which of the following activities would the nurse include in the teaching plan for a mother to help channel her 4-year-old child's energy? "

CHOICES

( X ) a.) Participation in parallel play.

( O ) b.) Play involving a game such as Simon Says.

( X ) c.) Bicycle riding.

( X ) d.) Stringing of large beads.


RATIONALE: A game such as Simon Says, which requires the preschooler to use a variety of motor skills, can help channel activity and meet developmental needs. Participation in parallel play is more appropriate for a younger child. Although the preschooler can ride a tricycle well, riding a bicycle requires more skill in balance than a 4-year-old child is likely to have. Stringing large beads is appropriate for a younger child. NURSING PROCESS STEP: Planning CLIENT NEEDS CATEGORY: Health promotion and maintenance CLIENT NEEDS SUBCATEGORY: None
******************************

--> QUESTION NUMBER _ 266 _ about (MC)


QUESTION: "The mother of a 4-year-old child asks about dental care for her child. "I help brush her teeth every day and her teeth look healthy. When should I take her to see a dentist?" Which of the following responses would be most appropriate?"

CHOICES

( X ) a.) "Because you help brush her teeth, there's no need to see a dentist right now."

( X ) b.) "Ideally she should have seen a dentist already, but it's still not too late."

( X ) c.) "Your child doesn't need to see the dentist until she starts school."

( O ) d.) "A dental checkup is a good idea even if no problems are noticeable."


RATIONALE: Routine dental examinations should begin when a child is young, usually after the age of 2 years, before any obvious problems develop. Even though the mother helps the child brush her teeth, teeth should be brushed after meals and at bedtime. Additionally, routine dental examinations should begin when a child is young, usually after the age of 2 years, before any obvious problems develop. Reprimanding the mother for not taking the child to the dentist is not therapeutic and may alienate the mother. Waiting until the child starts school may be too late because dental caries can occur before the age of 2 years. NURSING PROCESS STEP: Implementation CLIENT NEEDS CATEGORY: Health promotion and maintenance CLIENT NEEDS SUBCATEGORY: None
******************************

--> QUESTION NUMBER _ 267 _ about (MC)


QUESTION: "A mother tells the nurse that her 4 1/2-year-old child "doesn't seem to know the difference between right and wrong." The nurse responds to the mother basing the explanation on the fact that this behavior is typical of which of the following as described by Kohlberg's Theory of levels of moral development? "

CHOICES

( X ) a.) Autonomous.

( X ) b.) Conventional.

( O ) c.) Preconventional.

( X ) d.) Principled.


RATIONALE: The preconventional level of Kohlberg's stages of moral development is typical of the preschool-aged child. Stage I behaviors of this preconventional level have a punishment:obedience orientation. Children at this stage avoid punishment and avoid those who have power. Autonomous or postconventional is the third stage of moral development as described by Kohlberg. These children are concerned with defining values and principles. The conventional level of morality development pertains to children aged 7 to 12 years who are concerned with loyalty and conformity. Principled is another name for the autonomous or postconventional stage, the third stage level of moral development as described by Kohlberg. These children are concerned with defining values and principles. NURSING PROCESS STEP: Implementation CLIENT NEEDS CATEGORY: Health promotion and maintenance CLIENT NEEDS SUBCATEGORY: None
******************************

--> QUESTION NUMBER _ 268 _ about (MC)


QUESTION: "A mother tells the nurse that her other child, a 4-year-old boy, has developed some strange eating habits including not finishing meals and eating the same food for several days in a row. She would like to develop a plan to correct this situation. When developing such a plan, which of the following should the nurse and mother consider?"

CHOICES

( X ) a.) Deciding on a good reward for finishing the meal.

( O ) b.) Allowing him to make some decisions about the foods he eats.

( X ) c.) Restricting the availability of foods to those served at meal times.

( X ) d.) Not allowing him to leave the table until he has eaten the food.


RATIONALE: Allowing a child to make some decisions about the foods he eats and not insisting that he finish meals can avoid power struggles. Refusing to finish meals and to eat certain foods is normal behavior for a preschool-aged child. It is important to avoid tension at mealtime and to avoid confrontation about food, which can be used as a weapon, a bribe, or a pacifier. Rewarding a child for what is eaten can lead to power struggles between the parent and child over food. Restricting foods should be avoided; restriction can provoke power struggles and confrontation, thereby increasing tension. Not allowing the child to leave the table until finished can provoke power struggles and confrontation, thereby increasing tension. NURSING PROCESS STEP: Planning CLIENT NEEDS CATEGORY: Health promotion and maintenance CLIENT NEEDS SUBCATEGORY: None
******************************

--> QUESTION NUMBER _ 269 _ about (MC)


QUESTION: "Because both parents are nearsighted, the mother is concerned that her 4-year-old child may be nearsighted. She says that he likes to look at books and knows most of the alphabet. Which of the following assessment techniques should the nurse use to test the child's visual acuity?"

CHOICES

( X ) a.) Cross-over test.

( O ) b.) Allen picture cards.

( X ) c.) Snellen alphabet chart.

( X ) d.) Ishihara plates.


RATIONALE: Allen picture cards are used to test visual acuity in children who are not proficient with the alphabet. The cover and cross-over test is used to rule out strabismus. The Snellen alphabet chart is commonly used by age 8 or 9 years. Ishihara plates are used to test for color-blindness. NURSING PROCESS STEP: Assessment CLIENT NEEDS CATEGORY: Health promotion and maintenance CLIENT NEEDS SUBCATEGORY: None
******************************

--> QUESTION NUMBER _ 270 _ about (MC)


QUESTION: "Which of the following would be best to help prepare a preschool-aged child for an injection?"

CHOICES

( X ) a.) Having an older child explain that shots do not hurt.

( X ) b.) Helping the child to imagine she is in a different place.

( O ) c.) Giving the child a play syringe and a bandage to give a doll injections.

( X ) d.) Giving the child a pounding board to encourage expressions of anger.


RATIONALE: Allowing the preschool-aged child to give play injections can help to prepare the child to receive an injection. Preschoolers have a limited vocabulary. They express their feelings through play. They also use play to help cope with stress. Having an older child explain that shots do not hurt is inappropriate because preschool-aged children know that injections hurt. Misrepresenting would instill mistrust in the child. Imagery is appropriate with an older child during an injection. Using a pounding board after the experience can help the child express her feelings and feel in control again. NURSING PROCESS STEP: Implementation CLIENT NEEDS CATEGORY: Health promotion and maintenance CLIENT NEEDS SUBCATEGORY: None
******************************

--> QUESTION NUMBER _ 271 _ about (MC)


QUESTION: "When developing a teaching plan for a group of parents with 4-year-old children about the most effective strategies for safety, which of the following suggestions would the nurse expect to include?"

CHOICES

( X ) a.) Discussing the potential dangers to avoid.

( O ) b.) Modeling good examples of safe behavior.

( X ) c.) Showing pictures of children who have been involved in accidents.

( X ) d.) Calling the child "bad" when the child does something dangerous.


RATIONALE: Young children tend to imitate what they see, and parents teach by example intentionally or not. Thus, role modeling examples of safe behavior is key. Preschoolers model behavior that they see. Discussing potential dangers is too abstract for preschoolers to understand, thereby making it harder for them to realize the cause and effect. Using pictures of children involved in accidents would be more appropriate for an older child. As a child matures, parental interventions aimed at preventing accidents progress from protection to education. A child should not be labeled "bad" or "good" based on behavior; it is the behavior, not the child, that is undesirable. NURSING PROCESS STEP: Planning CLIENT NEEDS CATEGORY: Health promotion and maintenance CLIENT NEEDS SUBCATEGORY: None
******************************

--> QUESTION NUMBER _ 272 _ about (MC)


QUESTION: "When a father asks if his 5-year-old son should have the Denver Developmental Screening Test (DDST), which of the following information would the nurse need to obtain to respond to the father?"

CHOICES

( O ) a.) Father's understanding of the test.

( X ) b.) Father's sense of child's intelligence quotient (IQ).

( X ) c.) Child's developmental level.

( X ) d.) Child's previous test experience.


RATIONALE: Before the nurse responds to the father, the nurse needs to gather additional information about the father's knowledge and understanding of the test and the reason for the question. Once this information is obtained, the nurse can respond in the most appropriate manner. The child's IQ is not relevant to the father's question because the DDST does not measure IQ. The DDST is used to evaluate development in children from age 1 month to 6 years. Gathering information about the child's developmental level would be appropriate after the nurse determines the father's knowledge and understanding of the test and the reason for the question. The DDST would also provide information about the child's developmental level. Obtaining information about previous test experiences would be appropriate after the nurse determines the father's knowledge and understanding of the test and the reason for the question. NURSING PROCESS STEP: Assessment CLIENT NEEDS CATEGORY: Health promotion and maintenance CLIENT NEEDS SUBCATEGORY: None
******************************

--> QUESTION NUMBER _ 273 _ about (MC)


QUESTION: "A mother is concerned about her 9-year-old child's compulsion for collecting things. The nurse's explanation is based on the understanding that this behavior is related to the cognitive ability to perform which of the following?"

CHOICES

( O ) a.) Concrete operations.

( X ) b.) Formal operations.

( X ) c.) Coordination of secondary schemata.

( X ) d.) Tertiary circular reactions.


RATIONALE: The school-aged child (age 7 to 11 years) who has achieved the cognitive abilities required to master concrete operations often collects various objects when learning to manipulate and classify these objects. Formal operations do not emerge until later (age 11 to 15 years). Coordination of secondary schemata is part of the sensorimotor phase of cognitive development (up to age 2 years). Tertiary circular reactions are part of the sensorimotor phase of cognitive development (up to age 2 years). NURSING PROCESS STEP: Analysis CLIENT NEEDS CATEGORY: Health promotion and maintenance CLIENT NEEDS SUBCATEGORY: None
******************************

--> QUESTION NUMBER _ 274 _ about (MC)


QUESTION: "When developing a play program for a group of school-aged children by integrating Erikson's framework of psychosocial development, the nurse would incorporate play activities to aid in developing the child's sense of which of the following?"

CHOICES

( X ) a.) Initiative.

( O ) b.) Industry.

( X ) c.) Identity.

( X ) d.) Intimacy.


RATIONALE: According to Erikson, industry versus inferiority is the theme of psychosocial development during middle and late childhood. The challenge is mastering skills to create and complete projects; this is often done through play. Sense of initiative is the theme of the preschool-aged child. Sense of identity is the theme of early adolescence. Sense of intimacy and solidarity is the theme of late adolescence and young adulthood. NURSING PROCESS STEP: Planning CLIENT NEEDS CATEGORY: Health promotion and maintenance CLIENT NEEDS SUBCATEGORY: None
******************************

--> QUESTION NUMBER _ 275 _ about (MC)


QUESTION: "The parents of a 6-year-old child tell the nurse that they are concerned about the child's tonsils. On inspection, the nurse notes that the tonsils are large but not reddened or inflamed. The nurse interprets this finding as which of the following? "

CHOICES

( X ) a.) The need for tonsillectomy.

( X ) b.) An acute viral infection of the tonsils.

( O ) c.) A normal increase in lymphoid tissue.

( X ) d.) The need for an antibiotic.


RATIONALE: Because lymphoid tissue develops rapidly in relation to size until age 10 to 11 years, lymphoid hyperplasia in the form of enlarged tonsils is normal until age 6 to 7 years. After this time, the tissue slowly atrophies. Enlarged tonsils are not surgically removed unless they become abscessed or compromise physiologic functioning. Absence of redness and inflammation suggests the absence of infection. An antibiotic would be needed if the evidence suggested a bacterial infection. However, the tonsils are only enlarged, not reddened or inflamed, suggesting no infection. NURSING PROCESS STEP: Analysis CLIENT NEEDS CATEGORY: Health promotion and maintenance CLIENT NEEDS SUBCATEGORY: None
******************************

--> QUESTION NUMBER _ 276 _ about (MC)


QUESTION: "Which of the following preventive measures should the nurse stress during the first class on safety and accident prevention for a group of third graders?"

CHOICES

( X ) a.) Flame-retardant clothing.

( X ) b.) Life preservers.

( X ) c.) Protective eyewear.

( O ) d.) Automobile seat belts.


RATIONALE: Motor vehicle accidents are the most common cause of accidental injury and death in children between ages 1 and 12 years. Measures should be emphasized that prevent accidents involving motor vehicles, bicycles, or motorized bikes. Therefore, the use of automobile seat belts should be emphasized. Burns are a major cause of accidental injury in children under 1 year of age to school age, and flame-retardant clothing would be an appropriate safety strategy. However, because burns are not the most common cause, this topic could be dealt with after discussing motor vehicle safety. Drowning is a major cause of accidental injury and death in school-aged children, and the use of life preservers is appropriate. However, because drowning is not the most common cause, this topic could be dealt with after discussing motor vehicle safety. Eye injuries do occur, but they are not the most common cause of accidental injury and death in children ages 1 to 12 years of age. NURSING PROCESS STEP: Implementation CLIENT NEEDS CATEGORY: Safe, effective care environment CLIENT NEEDS SUBCATEGORY: Safety and infection control
******************************

--> QUESTION NUMBER _ 277 _ about (MC)


QUESTION: "When developing a plan of care with the mother who expresses concern that her 10-year-old son is overweight, the nurse would expect to include which of the following?"

CHOICES

( X ) a.) Eliminating the child's between-meal snacks.

( X ) b.) Eliminating the intake of fat from the diet.

( O ) c.) Including the child in meal planning and preparation.

( X ) d.) Allowing the child a daily intake of 1200 calories.


RATIONALE: Children ages 9 to 10 years can assume increasing responsibility for their health. Helping with meal preparation is an opportunity to learn about nutrition. Doing so also allows the child some control over the situation which enhances their compliance with the plan. Parents cannot continually monitor the school-aged child's food intake because the child's world is expanding. Thus, it would be difficult to eliminate the child's between-meal snacks. To continue to grow, children need fat in their diet. Also, it would be extremely difficult to eliminate all fat from the child's diet. A school-aged child requires about 2400 calories per day. NURSING PROCESS STEP: Planning CLIENT NEEDS CATEGORY: Health promotion and maintenance CLIENT NEEDS SUBCATEGORY: None
******************************

--> QUESTION NUMBER _ 278 _ about (MC)


QUESTION: "When planning counseling sessions with adolescents, the nurse must incorporate measures to consider which of the following abilities?"

CHOICES

( X ) a.) Assimilation and accommodation.

( O ) b.) Capacity to deal with abstract possibilities.

( X ) c.) Representational thought.

( X ) d.) Conservation problem solving.


RATIONALE: The ability to deal with abstract possibilities develops during adolescence. Assimilation and accommodation are characteristics of an infant's sensorimotor development. Representational thought is associated with the preconceptual phase of development, from the ages of 2 to 4 years. Problems of conservation are part of concrete operations learned by children between ages 4 and 7 years. NURSING PROCESS STEP: Planning CLIENT NEEDS CATEGORY: Health promotion and maintenance CLIENT NEEDS SUBCATEGORY: None
******************************

--> QUESTION NUMBER _ 279 _ about (MC)


QUESTION: "A 16-year-old girl comes to the school nurse complaining of cramps, backache, and nausea with her periods. The nurse most likely would interpret these symptoms as which of the following?"

CHOICES

( X ) a.) Pathologic.

( O ) b.) Physiologic.

( X ) c.) Psychogenic.

( X ) d.) Psychosomatic.


RATIONALE: The basis for these symptoms is most likely physiologic. There are two types of dysmenorrhea: primary and secondary. Primary, the most common type, is believed to be caused by an increased level of prostaglandins producing uterine hyperactivity and contractions. Complaints of nausea, cramps, and backache with a female adolescent's menstrual periods are considered a normal, not pathologic, physiologic response. Complaints of nausea, cramps, and backache with a female adolescent's menstrual periods are considered a normal physiologic, not psychogenic, response. Complaints of nausea, cramps, and backache with a female adolescent's menstrual periods are considered a normal physiologic, not psychosomatic, response. NURSING PROCESS STEP: Analysis CLIENT NEEDS CATEGORY: Health promotion and maintenance CLIENT NEEDS SUBCATEGORY: None
******************************

--> QUESTION NUMBER _ 280 _ about (MC)


QUESTION: "A mother asks the school nurse if her 16-year-old son still needs immunizations. Which of the following explanations would be most appropriate?"

CHOICES

( X ) a.) Children older than age 7 years do not need immunizations.

( X ) b.) Adolescents should routinely receive a measles vaccination at age 16 years.

( X ) c.) The last immunization received is a tetanus booster at age 16 years.

( O ) d.) Adolescents should receive a tetanus diphtheria booster every 10 years.


RATIONALE: The American Academy of Pediatrics, Committee on Infectious Diseases (1999), recommends routine immunization with a combined tetanus toxoid and diphtheria toxoid booster every 10 years for adolescents and adults. The American Academy of Pediatrics, Committee on Infectious Diseases (1999), recommends routine immunization with a combined tetanus toxoid and diphtheria toxoid booster every 10 years for adolescents and adults. Measles vaccines are to be repeated at 4 to 6 years of age or on entering sixth grade. Measles vaccination should not be routinely given to adolescent girls because of the possible fetal effects if the girl is pregnant. A combined tetanus toxoid and diphtheria toxoid booster is recommended every 10 years for adolescents and adults. NURSING PROCESS STEP: Implementation CLIENT NEEDS CATEGORY: Health promotion and maintenance CLIENT NEEDS SUBCATEGORY: None
******************************

--> QUESTION NUMBER _ 281 _ about (MC)


QUESTION: "The parents of teenagers express concerns about the types and large quantities of food their children eat and their refusal to eat foods served at family meals. Which of the following suggestions would be most helpful for the parents?"

CHOICES

( X ) a.) Carefully evaluate the adolescents' nutritional intake.

( X ) b.) Inform the adolescents about the adverse effects of fad diets.

( X ) c.) Give the adolescents responsibility for grocery shopping for 1 month.

( O ) d.) Incorporate the adolescents' preferences into meal planning.


RATIONALE: Preventing food intake from becoming the center of an independence:dependence struggle is important. Nursing responsibilities include helping parents realize that adolescents require a high caloric intake and they need to make individual decisions. Adolescents are subject to peer pressure that often supersedes family pressure. The parents have already evaluated their adolescents' diet, as evidenced by their concerns. Although the adolescent should be informed about the adverse effects of fad diets, doing so does not ensure the adolescent's adequate nutritional intake. Responsibility for grocery shopping for a month may encourage independence but does not ensure adequate nutritional status. NURSING PROCESS STEP: Implementation CLIENT NEEDS CATEGORY: Health promotion and maintenance CLIENT NEEDS SUBCATEGORY: None
******************************

--> QUESTION NUMBER _ 282 _ about (MC)


QUESTION: "An 18-year-old senior tells the nurse, "Everyone does it, so it's all right," to justify rule-breaking behavior. The nurse realizes this is an example of which level or stage of moral reasoning development as described by Kohlberg?"

CHOICES

( X ) a.) Preconventional.

( O ) b.) Conventional.

( X ) c.) Postconventional.

( X ) d.) Autonomous



RATIONALE: Stage three behaviors of Kohlberg's conventional level of moral reasoning focus on the approval of others. Moral dilemmas are solved by the group standard, with an emphasis on conformity. Adolescents usually function at this level of moral development. Children up to age 2 years function at the preconventional level that is characterized by avoiding punishment and obeying those who have power. Children 11 years of age and older and adults function at the postconventional level or autonomous stage, which is characterized by defining moral values and understanding that correct behavior is determined by society. Children 11 years of age and older and adults function at the postconventional level or autonomous stage, which is characterized by defining moral values and understanding that correct behavior is determined by society. NURSING PROCESS STEP: Analysis CLIENT NEEDS CATEGORY: Health promotion and maintenance CLIENT NEEDS SUBCATEGORY: None
******************************

--> QUESTION NUMBER _ 283 _ about (MC)


QUESTION: "As part of the annual health screening, the nurse visits the eighth-grade physical education classes and asks each student to bend forward at the waist with the back parallel to the floor and the hands together at midline. For which of the following is the nurse assessing?"

CHOICES

( X ) a.) Slipped epiphysis.

( X ) b.) Developmental dysplasia of hip.

( O ) c.) Idiopathic scoliosis.

( X ) d.) Physical dexterity.


RATIONALE: When bending forward, a person who has idiopathic scoliosis has an obvious rib hump. The two sides of the back at the hips, ribs, or shoulders are not level. Slipped epiphysis is characterized by continuous or intermittent hip pain and a tendency of the leg to rotate externally. The nurse assesses for this by determining range of motion of the hip. Developmental dysplasia of the hip is an abnormality of the hip joint at birth that is evaluated by the Ortolani or Barlow's test. Physical dexterity is assessed by obtaining a history about what the child can do, such as riding a bicycle, putting a puzzle together, and keeping up with other children of the same age. Asking a child to bend over and bring hands to the midline does not provide information about the child's physical dexterity. NURSING PROCESS STEP: Assessment CLIENT NEEDS CATEGORY: Health promotion and maintenance CLIENT NEEDS SUBCATEGORY: None
******************************

--> QUESTION NUMBER _ 284 _ about (MC)


QUESTION: "A parent says that her family will soon be traveling abroad and asks why the drinking water in many regions must be boiled. The nurse would explain that, in addition to various types of dysentery, contaminated drinking water is most commonly responsible for the transmission of which of the following?"

CHOICES

( X ) a.) Yellow fever.

( X ) b.) Brucellosis.

( X ) c.) Poliomyelitis.

( O ) d.) Typhoid fever.


RATIONALE: Water is the usual vehicle for spreading typhoid fever. Yellow fever is spread through insect bites. Brucellosis (undulant fever) is spread by contaminated cow's milk. Poliomyelitis is most probably spread through respiratory secretions. NURSING PROCESS STEP: Implementation CLIENT NEEDS CATEGORY: Health promotion and maintenance CLIENT NEEDS SUBCATEGORY: None
******************************

--> QUESTION NUMBER _ 285 _ about (MC)


QUESTION: "Which of the following home regimens should the nurse suggest to relieve itching in children with chicken pox?"

CHOICES

( X ) a.) Generous amounts of fine baby powder.

( O ) b.) Oatmeal preparation baths.

( X ) c.) Terrycloth towels moistened with hydrogen peroxide.

( X ) d.) Cool compresses moistened with a weak salt solution.


RATIONALE: Because of colloidal properties, oatmeal preparation baths often help relieve the itching associated with chicken pox. Calamine lotion can be used also. Baby powder is unlikely to relieve itching because it acts primarily to absorb moisture. A terrycloth towel moistened with hydrogen peroxide is unlikely to relieve itching. Rather, hydrogen peroxide is used to clean wounds. A cool compress moistened with a weak salt solution is unlikely to relieve itching because it does not have any antipruritic properties. NURSING PROCESS STEP: Implementation CLIENT NEEDS CATEGORY: Physiological integrity CLIENT NEEDS SUBCATEGORY: Basic care and comfort
******************************

--> QUESTION NUMBER _ 286 _ about (MC)


QUESTION: "A parent of a child with hemophilia states that she worries whenever the child has a bump or cut. The nurse should explain that after the area is cleansed, the wound should be treated by applying which of the following?"

CHOICES

( O ) a.) Gentle pressure.

( X ) b.) Warm, moist compresses.

( X ) c.) Tourniquet above the injured area.

( X ) d.) A wet-to-dry dressing.


RATIONALE: In children with hemophilia (an inherited bleeding disorder), a bump, bruise, or cut can cause serious bleeding. After the injured area is cleansed, gentle pressure should be applied to allow clot formation to help stop the bleeding. In addition, the area should be immobilized and elevated. Cold applications, instead of warm moist compresses, are often used to promote vasoconstriction and help control the bleeding. A tourniquet should not be used because of the high risk of tissue hypoxia and resulting necrosis. Wet-to-dry dressings should be avoided because they could be irritating to the area. NURSING PROCESS STEP: Implementation CLIENT NEEDS CATEGORY: Physiological integrity CLIENT NEEDS SUBCATEGORY: Reduction of risk potential
******************************

--> QUESTION NUMBER _ 287 _ about (MC)


QUESTION: "Which of the following over-the-counter medications would the nurse warn the mother of a child with hemophilia to avoid using?"

CHOICES

( O ) a.) Acetylsalicylic acid.

( X ) b.) Magnesium hydroxide.

( X ) c.) Acetaminophen.

( X ) d.) Multivitamin capsules.


RATIONALE: Acetylsalicylic acid (aspirin) inhibits platelet aggregation, prolongs bleeding time, and inhibits prothrombin synthesis. It is, therefore, contraindicated for a child with hemophilia. Magnesium hydroxide (Milk of Magnesia) has no effect on bleeding and is not contraindicated for clients with hemophilia. Acetaminophen (Tylenol) is the recommended alternative for analgesic and antipyretic purposes. Multivitamin capsules have no effect on bleeding and are not contraindicated for clients with hemophilia. NURSING PROCESS STEP: Implementation CLIENT NEEDS CATEGORY: Physiological integrity CLIENT NEEDS SUBCATEGORY: Pharmacological and parenteral therapies
******************************

--> QUESTION NUMBER _ 288 _ about (MC)


QUESTION: "Which of the following suggestions would be most appropriate in helping parents to prepare their children for starting school?"

CHOICES

( X ) a.) Have an older sibling tell the child about school.

( O ) b.) Orient the child to the school's physical environment.

( X ) c.) Offer to stay with the child for the first few days of school.

( X ) d.) Discuss school with the child if he or she asks about it.


RATIONALE: To help prepare a child to enter school, it is generally recommended that the child be taken to school to become oriented to the physical surroundings. This helps to lessen the child's concerns about what the school looks like and where he or she is going, thereby helping to alleviate some fears of the unknown. Older siblings are likely to criticize the younger child, possibly adding to the child's fears and feelings of insecurity. Staying with the child for a few days is not advised because it would foster dependence. The child needs to develop independence. The child may ask questions out of fear. Therefore, good preparation probably cannot be accomplished through discussions. NURSING PROCESS STEP: Planning CLIENT NEEDS CATEGORY: Health promotion and maintenance CLIENT NEEDS SUBCATEGORY: None
******************************

--> QUESTION NUMBER _ 289 _ about (MC)


QUESTION: "The nurse should explain that the most common cause for the unhappiness some children experience when first entering school is due to which of the following?"

CHOICES

( O ) a.) Feelings of insecurity.

( X ) b.) Social isolation.

( X ) c.) Emotional maladjustment.

( X ) d.) Poor language development.


RATIONALE: The child entering school is moving into a new environment after having experienced security at home. Unhappiness with resulting feelings of insecurity is a normal response to the lost sense of security. Social isolation suggests a psychosocial disturbance and should not be playing a role among normal children who seem unhappy about entering school. Emotional maladjustment suggests a psychosocial disturbance and should not be playing a role among normal children who seem unhappy about entering school. Poor language development suggests a psychosocial disturbance and should not be playing a role among normal children who seem unhappy about entering school. NURSING PROCESS STEP: Implementation CLIENT NEEDS CATEGORY: Health promotion and maintenance CLIENT NEEDS SUBCATEGORY: None
******************************

--> QUESTION NUMBER _ 290 _ about (MC)


QUESTION: "When discussing adolescent behavior with the parents, the nurse would incorporate information about an adolescent's need to develop a sense of which of the following?"

CHOICES

( O ) a.) Identity.

( X ) b.) Industry.

( X ) c.) Initiative.

( X ) d.) Intimacy.


RATIONALE: According to Erikson's theory, the central problem confronting adolescents is establishing a sense of identity. School-aged children focus on attaining a sense of industry. Preschool-aged children are confronted with establishing a sense of initiative. The core problem of young adulthood is concerned with intimacy. NURSING PROCESS STEP: Implementation CLIENT NEEDS CATEGORY: Health promotion and maintenance CLIENT NEEDS SUBCATEGORY: None
******************************

--> QUESTION NUMBER _ 291 _ about (MC)


QUESTION: "Which of the following substances would the nurse include in the teaching plan for a group of parents of children with acne requesting information about cleansing the affected skin?"

CHOICES

( X ) a.) Witch hazel.

( O ) b.) Soap and water.

( X ) c.) Hydrogen peroxide.

( X ) d.) Lotions and creams.


RATIONALE: Acne is a disorder of the pilosebaceous follicles (hair follicles and sebaceous gland complex). During adolescence, the secretions of the sebaceous glands increase, altering the follicular lining and causing occlusion of the ducts with accumulated sebum. Bacteria in the follicle then cause an infection. Frequent washing of affected areas with soap and water is recommended to act as a mild peeling agent and reduce secondary infection. Witch hazel is an astringent that can be used after thoroughly cleansing the skin. Hydrogen peroxide is a poor cleansing agent for skin with acne. Lotions and creams aggravate the condition by adding more oily substances to the already oily skin. NURSING PROCESS STEP: Planning CLIENT NEEDS CATEGORY: Physiological integrity CLIENT NEEDS SUBCATEGORY: Physiological adaptation
******************************

--> QUESTION NUMBER _ 292 _ about (MC)


QUESTION: "Which of the following would the nurse expect to include in a community health program designed to control sexually transmitted diseases (STDs)?"

CHOICES

( X ) a.) Mass screening of all individuals.

( O ) b.) Location of the possible sources of infection.

( X ) c.) Treatment of those with the disease.

( X ) d.) Isolation of those suspected of having STDs.


RATIONALE: Community health measures for controlling STDs most commonly focus on locating the sources of infection. Doing so allows the infected person to identify the person's sexual contacts and urge them to get treatment. Although STDs are prevalent, mass screening for all individuals is impractical. Community health measures for controlling STDs most commonly focus on locating the sources of infection. Doing so allows the infected person to identify the person's sexual contacts and urge them to get treatment. Treatment can occur only if the person requests it. It would be impractical to isolate persons suspected of having STDs. The priority is to identify persons with STDs so they can be treated. NURSING PROCESS STEP: Planning CLIENT NEEDS CATEGORY: Health promotion and maintenance CLIENT NEEDS SUBCATEGORY: None
******************************

--> QUESTION NUMBER _ 293 _ about (MC)


QUESTION: "A 4-year-old child asks the nurse if it will hurt to have the tonsils and adenoids taken out. Which of the following responses by the nurse would be best?"

CHOICES

( X ) a.) "It won't hurt because we put you to sleep."

( X ) b.) "It won't hurt because you're such a big boy."

( X ) c.) "It will hurt because of the incisions made in the throat."

( O ) d.) "It will hurt, but we have medicine to help you feel better."


RATIONALE: Preschool-aged children are fearful of physical injury. Truthful but simple explanations will minimize distorted fears and reduce anxiety. Telling the child that it won't hurt is inappropriate, not truthful, and can possibly lead to mistrust. Telling the child that it won't hurt is inappropriate and untrue. Additionally, stating that the client is a big boy may deter the child from reporting pain for fear of being labeled a 'baby.' A detailed explanation may be beyond the child's understanding and add to his fears. Using the term incision is inappropriate because the child probably will not understand what this term means. NURSING PROCESS STEP: Implementation CLIENT NEEDS CATEGORY: Health promotion and maintenance CLIENT NEEDS SUBCATEGORY: None
******************************

--> QUESTION NUMBER _ 294 _ about (MC)


QUESTION: "Preoperatively, the nurse discusses with the 4-year-old child and parents the plan of care that will be implemented when the child returns from the post-anesthesia care unit (PACU). Which of the following interventions should the nurse emphasize?"

CHOICES

( X ) a.) Need for frequent coughing.

( X ) b.) Use of acetylsalicylic acid for pain, as needed.

( X ) c.) Ability to have ice cream right after surgery.

( O ) d.) Use of sips of clear liquids when awake and alert.


RATIONALE: Once the child is alert, he may have sips of clear liquids. Eating enhances the blood supply to the throat, which promotes rapid healing. However, the child should start with clear fluids. Coughing is discouraged because it disrupts the suture line and may cause bleeding. Acetylsalicylic acid is contraindicated because it interferes with platelet aggregation and promotes bleeding. Once the child is able to tolerate clear liquids, he or she can progress to a full liquid diet that would include ice cream. NURSING PROCESS STEP: Implementation CLIENT NEEDS CATEGORY: Physiological integrity CLIENT NEEDS SUBCATEGORY: Reduction of risk potential
******************************

--> QUESTION NUMBER _ 295 _ about (MC)


QUESTION: "Upon the child's return from the post-anesthesia recovery unit (PACU) after a tonsillectomy, the nurse would place the child in which of the following positions?"

CHOICES

( X ) a.) Trendelenburg.

( X ) b.) Supine.

( O ) c.) Prone.

( X ) d.) Lithotomy.


RATIONALE: Placing the child in a prone or a side-lying position facilitates drainage of secretions and helps prevent aspiration. The Trendelenburg position is contraindicated because it decreases effective lung volumes. The supine position is contraindicated because of the increased risk of aspiration. The lithotomy position is used for a pelvic examination. NURSING PROCESS STEP: Implementation CLIENT NEEDS CATEGORY: Physiological integrity CLIENT NEEDS SUBCATEGORY: Reduction of risk potential
******************************

--> QUESTION NUMBER _ 296 _ about (MC)


QUESTION: "Which of the following would be the best to offer first to a 4-year-old child who has had a tonsillectomy?"

CHOICES

( O ) a.) Yellow popsicle.

( X ) b.) Chocolate milk.

( X ) c.) Cherry fruit drink.

( X ) d.) Vanilla pudding.


RATIONALE: The nurse must consider both the color and consistency of foods and fluids given initially. Red or brown foods and fluids should be avoided so that if vomiting occurs, fresh or old blood can be distinguished from the ingested liquids. Clear liquids are offered first. Therefore, a yellow popsicle would be best. The nurse must consider both the color and consistency of foods and fluids given initially. Red or brown foods and fluids should be avoided so that if vomiting occurs, fresh or old blood can be distinguished from the ingested liquids. Chocolate milk, because of its color and also because it is considered a full liquid, would not be an appropriate choice. The nurse must consider both the color and consistency of foods and fluids given initially. Red or brown foods and fluids should be avoided so that if vomiting occurs, fresh or old blood can be distinguished from the ingested liquids. Cherry fruit drink, because of its red color, would be an inappropriate choice. The nurse must consider both the color and consistency of foods and fluids given initially. Red or brown foods and fluids should be avoided so that if vomiting occurs, fresh or old blood can be distinguished from the ingested liquids. Vanilla pudding, although an appropriate color, is considered part of a full liquid diet and should not be offered until the child can retain clear liquids. NURSING PROCESS STEP: Implementation CLIENT NEEDS CATEGORY: Physiological integrity CLIENT NEEDS SUBCATEGORY: Physiological adaptation
******************************

--> QUESTION NUMBER _ 297 _ about (MC)


QUESTION: "Which of the following signs and symptoms, if verbalized by the parents of a child who is being discharged after a tonsillectomy, would indicate to the nurse that the parents understand when they should seek medical attention?"

CHOICES

( X ) a.) Low-grade fever.

( O ) b.) Frequent swallowing.

( X ) c.) Slight ear pain.

( X ) d.) Objectionable mouth odor.


RATIONALE: Frequent swallowing indicates that the child is probably swallowing bloody drainage from the tonsillar beds. Thus, the physician should be notified. Most likely, the child will have a low-grade fever in response to the surgery. Slight pain from the surgical site is often referred to the ear. Objectionable mouth odor is common secondary to the presence of some old blood after this surgical procedure. NURSING PROCESS STEP: Evaluation CLIENT NEEDS CATEGORY: Physiological integrity CLIENT NEEDS SUBCATEGORY: Reduction of risk potential
******************************

--> QUESTION NUMBER _ 298 _ about (MC)


QUESTION: "When the nurse is teaching a group of parents about common childhood problems, a parent asks, "Why are children more likely to develop ear infections than adults are?" The nurse bases the response to this question on the understanding that the key anatomic difference between adults and children is due to which of the following structures?"

CHOICES

( X ) a.) Nasopharynx.

( O ) b.) Eustachian tubes.

( X ) c.) Ear canals.

( X ) d.) Tympanic membranes.


RATIONALE: In infants and young children, the eustachian tubes are short and lie in a relatively horizontal position. This anatomic position favors the development of otitis media because it is easy for materials from the nasopharynx to enter the tubes. Although bacteria may be present in the nasopharynx, this does not affect middle ear function. The size of the ear canal has no impact on the increased number of ear infections in children. Rather, in infants and young children, the eustachian tubes are short and lie in a relatively horizontal position. This anatomic position favors the development of otitis media because it is easy for materials from the nasopharynx to enter the tubes. An intact tympanic membrane prevents bacteria from entering the middle ear from the external ear canal. The tympanic membrane changes appearance with an ear infection, but its structure does not predispose infants and young children to ear infection. NURSING PROCESS STEP: Analysis CLIENT NEEDS CATEGORY: Health promotion and maintenance CLIENT NEEDS SUBCATEGORY: None
******************************

--> QUESTION NUMBER _ 299 _ about (MC)


QUESTION: "The parents report that the child has a runny nose, fever, cough, and is irritable and constantly rubbing his ears. The nurse would expect to see a tympanic membrane that appears as which of the following?"

CHOICES

( O ) a.) Bulging and red.

( X ) b.) Clear and inverted.

( X ) c.) Pearly gray.

( X ) d.) Scarred.


RATIONALE: Based on the report of the child's signs and symptoms, the nurse would suspect otitis media. On assessment, the tympanic membrane would appear bulging and bright red (because of increased middle ear pressure), typically indicative of otitis media. Other characteristic findings include rhinorrhea, fever, cough, irritability, pulling at the ears, earache, vomiting, and diarrhea. A reddened, nonbulging tympanic membrane may indicate otitis media if the membrane has ruptured. A clear, inverted membrane may indicate a blockage of the eustachian tubes. A pearly gray tympanic membrane is normal. A scarred tympanic membrane indicates that the membrane has burst due to pressure, but this condition would have occurred earlier if scar tissue has formed. NURSING PROCESS STEP: Assessment CLIENT NEEDS CATEGORY: Physiological integrity CLIENT NEEDS SUBCATEGORY: Physiological adaptation
******************************

--> QUESTION NUMBER _ 300 _ about (MC)


QUESTION: "After teaching the mother of an 18-month old child being discharged with a prescription for co-trimoxazole (Bactrim), which of the following statements by the mother would indicate the need for additional teaching?"

CHOICES

( X ) a.) "I will watch to see that he is wetting his diapers as usual."

( O ) b.) "I should give the medicine after he eats some food."

( X ) c.) "If he develops a sunburn-type rash, I will stop the medication and call the physician."

( X ) d.) "I will make sure he drinks plenty of extra fluids."


RATIONALE: When co-trimoxazole (Bactrim) is ordered, it should be given on an empty stomach to promote absorption, thereby maximizing its effectiveness. Thus, the mother would need additional instruction if she stated she was to give the medication with food. Monitoring the child's urinary output by observing the child's diapers is important because diminished renal function is a possible side effect. The child needs to be monitored for Steven-Johnson syndrome, a possible and serious side effect that causes a sunburn-type rash. If it develops, the medication should be held and the health care provider should be notified. Drinking extra fluids is essential to promote adequate renal function for the child receiving a sulfa-type antibiotic. NURSING PROCESS STEP: Evaluation CLIENT NEEDS CATEGORY: Physiological integrity CLIENT NEEDS SUBCATEGORY: Pharmacological and parenteral therapies
******************************